Jacobian transformation, find new limits

Click For Summary
The discussion focuses on finding the limits for U and V in a Jacobian transformation involving the equations x - 2y = 0, x - 2y = 4, 3x - y = 1, and 3x - y = 8. Users express confusion over determining the correct limits for U and V, despite attempting to analyze the maximum and minimum coordinates from the graphs. The transformation results in limits of u = 0, u = 4, v = 1, and v = 8, which are derived from the boundaries of the original equations. It is suggested that sketching the region can clarify the relationship between u and v in the xy-plane. Understanding the boundaries of the transformation is crucial for accurately determining the limits.
Feodalherren
Messages
604
Reaction score
6

Homework Statement


jacobian.png



Homework Equations





The Attempt at a Solution


What I don't understand is how I'm supposed to find those limits for U and V. That's not at all what I'm getting. I've tried solving for the max and min x,y coordinates from the given graphs but that doesn't yield the correct answer.
 
Physics news on Phys.org
Look at the original equations you are given:

##x - 2y = 0##
##x - 2y = 4##
##3x - y = 1##
##3x - y = 8##

Applying your transformation yields:

##u = 0##
##u = 4##
##v = 1##
##v = 8##

These yield your limits.
 
  • Like
Likes 1 person
It quite often helps to make a sketch of the region. What do the u=constant and v=constant lines look like in the xy-plane?
 
Feodalherren said:
What I don't understand is how I'm supposed to find those limits for U and V. That's not at all what I'm getting. I've tried solving for the max and min x,y coordinates from the given graphs but that doesn't yield the correct answer.
It unfortunate that you did not tell us how you tried "solving for the max and min x, y coordinates" since if you had we might be able to point out your mistake. All I can say is that if you have "u= x- 2y", one of the boundaries is x- 2y= 0 and the other is x- 2y= 4, u= 0 and 4 pretty much leaps out at you- u= x- 2y= 0 and u= x- 2y= 4!
 
Question: A clock's minute hand has length 4 and its hour hand has length 3. What is the distance between the tips at the moment when it is increasing most rapidly?(Putnam Exam Question) Answer: Making assumption that both the hands moves at constant angular velocities, the answer is ## \sqrt{7} .## But don't you think this assumption is somewhat doubtful and wrong?

Similar threads

Replies
21
Views
2K
  • · Replies 16 ·
Replies
16
Views
2K
  • · Replies 2 ·
Replies
2
Views
2K
  • · Replies 5 ·
Replies
5
Views
2K
  • · Replies 4 ·
Replies
4
Views
2K
  • · Replies 14 ·
Replies
14
Views
2K
  • · Replies 8 ·
Replies
8
Views
2K
  • · Replies 4 ·
Replies
4
Views
3K
  • · Replies 3 ·
Replies
3
Views
2K
  • · Replies 7 ·
Replies
7
Views
5K